Find the slope of the line on the graph.
Write your answer as a fraction or a whole
number, not a mixed number or decimal.

Find The Slope Of The Line On The Graph.Write Your Answer As A Fraction Or A Wholenumber, Not A Mixed

Answers

Answer 1

Answer:

-3

Step-by-step explanation:

We can see that the line passes through (0,3) and (1,0)

So using rise over run we have

-3/1

or

-3


Related Questions

A shop sells boxes of chocolate. In total there are 252 dark chocolate and 180 milk chocolates.if every box is identical,how many boxes could there be?

Answers

Answer:

12 boxes.

Step-by-step explanation:

We need to find the greatest common factor of 251 and 180:

First find the prime factors :

252 = 2*2*3*3*7

180 =  2*2*3*3*5

So the GCF = 2*2*3*3 = 36.

So id each box is identical, there could be 7 boxes of dark chocolates and 5 boxes of milk chocolates each containing 36 chocolates.

Total number of boxes = 12.

An archaeologist locates a fossil of an early human skeleton. To determine the age of the fossil, the archaeologist utilizes a technique called carbon dating, where the relative amount of carbon-14 can help determine the age of the fossil. Carbon-14 has a half-life of about 5700 years. He finds that the fossil contains 15% of the amount of carbon-14 anticipated when compared to a living femur of the same size. The decay of carbon-14 can be calculated as shown below, where N0 is original amount of carbon-14, t is the time of decay, in years, r represents the rate of decay, and N(t) represents the amount of carbon-14 remaining. Rounded to the nearest year, the skeleton is approximately years old.

Answers

Answer:

The skeleton is approximately 18970 years old.

Step-by-step explanation:

The amount of Carbon-14 remaining after t years is given by the following equation:

[tex]N(t) = N(0)(1-r)^{t}[/tex]

In which N(0) is the initial amount and r is the decay rate, as a decimal.

Carbon-14 has a half-life of about 5700 years.

This means that [tex]N(5700) = 0.5N(0)[/tex]

So

[tex]N(t) = N(0)(1-r)^{t}[/tex]

[tex]0.5N(0) = N(0)(1-r)^{5700}[/tex]

[tex](1-r)^{5700} = 0.5[/tex]

[tex]\sqrt[5700]{(1-r)^{5700}} = \sqrt[5700]{0.5}[/tex]

[tex]1-r = 0.9999[/tex]

[tex]r = 1 - 0.9999[/tex]

[tex]r = 0.0001[/tex]

So

[tex]N(t) = N(0)(1-r)^{t}[/tex]

[tex]N(t) = N(0)(0.9999)^{t}[/tex]

He finds that the fossil contains 15% of the amount of carbon-14 anticipated when compared to a living femur of the same size.

The age is t for which N(t) = 0.15N(0). So

[tex]N(t) = N(0)(0.9999)^{t}[/tex]

[tex]0.15N(0) = N(0)(0.9999)^{t}[/tex]

[tex](0.9999)^{t} = 0.15[/tex]

[tex]\log{(0.9999)^{t}} = \log{0.15}[/tex]

[tex]t\log{0.9999} = \log{0.15}[/tex]

[tex]t = \frac{\log{0.15}}{\log{0.9999}}[/tex]

[tex]t = 18970.25[/tex]

Rounding to the nearest year.

The skeleton is approximately 18970 years old.

Answer:

Mark as Brainliest Please :)

Step-by-step explanation:

A board game has a spinner divided into sections of equal size. Each section is labeled with a number between 1 and 5. Which number is a reasonable estimate of the number of times the spinner will land on a section labeled with a 5 over the course of 300 spins?

Answers

Answer:

75

Step-by-step explanation:

first, there are three 5's on the spinner, with a total of 12 sections. 3/12 = 0.25, so you multiply 0.25 times 300 (the total spins) and get 75

If a pizza is cut into 6 slices, what is the angle measure for each slice?

Answers

A whole circle is 360 degrees so just 360:6 so 60 degree

QUESTION 4
What kind of passenger would be considered a false negative?
A) Someone with dangerous material that pass through security
B) Someone with dangerous material thpt is stopped by security
C) Someone without dangerous material that pass through security
D) Someone without dangerous material that is stopped by security

Answers

I think D is the correct answer

Riverside Elementary School is holding a school-wide election to choose a school color. 5/8 of the votes were for blue, 5/9 of the remaining votes were for green 48 votes were for red. How many votes were for blue? how many votes were for green? if every student got one vote, but there were 25 students absent on the day of the vote, how many students are there at Riverside Elementary School

Answers

Answer:

288 votes

Step-by-step explanation:

wright out what you know

Blue: 5/8

Green: 5/9 (of 3/8 because it is the remaining parts) so 5/24

Red: 1 - (5/8 + 5/24) = 1/6

Find the total amount of people

You know that 1/6 of the population is 48, so the total population is 288 (48×6)

Multiply the total population by the fractions to get the number of votes

Which of the following is graphed below

Answers

Answer:

The answer is C

What is the value of y?
-36
-108
-72
-54

Answers

Answer:

54°

Step-by-step explanation:

The overall angle degrees of a trinangle 180°

Since we know that once of the angles is 72° we subract that from 180°

That equals 108°

Then we divide 108° since that is the sum of both the angles.

108/2= 54, so y= 54°

if u can do all 3 that will be great, TYSM!!

Answers

A: Five people each paying $14.50 means that the total bill was [tex]5\cdot\$14.50=\boxed{\$72.50}.[/tex]

B: Subtract the number of people he already has from the number of people he needs to get [tex]42-29=\boxed{13}[/tex] people.

C: Divide 96 by 8 to get [tex]\boxed{12}[/tex] packs.

Arithmetic is where it all begins, y'know?

Answer:

Hope this helps though I m not sure about the first one since I got another answer 72.5 but it isn't there

Television viewing reached a new high when the global information and measurement company reported a mean daily viewing time of hours per household. Use a normal probability distribution with a standard deviation of hours to answer the following questions about daily television viewing per household.
a. what is the probability that a household views television between 6 and 8 hours a day (to 4 decimals)?
b. How many hours of television viewing must a household have in order to be in the top 5% of all television viewing households (to 2 decimals)?
c. What is the probability that a household views television more than 5 hours a day (to 4 decimals)?

Answers

Answer:

Step-by-step explanation:

The question is incomplete. The complete question is:

Television viewing reached a new high when the global information and measurement company reported a mean daily viewing time of 8.35 hours per household. Use a normal probability distribution with a standard deviation of 2.5 hours to answer the following questions about daily television viewing per household.

(a.) what is the probability that a household views television between 6 and 8 hours a day (to 4 decimals)?

(b.) How many hours of television viewing must a household have in order to be in the top 5% of all television viewing households (to 2 decimals)?

(c.) What is the probability that a household views television more than 5 hours a day (to 4 decimals)?

Solution:

Let x be the random variable representing the television viewing times per household. Since it is normally distributed and the population mean and population standard deviation are known, we would apply the formula,

z = (x - µ)/σ

Where

x = sample mean

µ = population mean

σ = standard deviation

From the information given,

µ = 8.35

σ = 2.5

a) the probability that a household views television between 6 and 8 hours a day is expressed as

P(6 ≤ x ≤ 8)

For x = 6,

z = (6 - 8.35)/2.5 = - 0.94

Looking at the normal distribution table, the probability corresponding to the z score is 0.1736

For x = 8

z = (8 - 8.35)/2.5 = - 0.14

Looking at the normal distribution table, the probability corresponding to the z score is 0.4443

Therefore,

P(6 ≤ x ≤ 8) = 0.4443 - 0.1736 = 0.2707

b) the top 5% means greater than 95%. It means that the sample mean is greater than the population mean and the z score is positive. The corresponding z score from the normal distribution table is 1.645. Therefore,

(x - 8.35)/2.5 = 1.645

Cross multiplying, it becomes

x - 8.35 = 2.5 × 1.645 = 4.11

x = 4.11 + 8.35 = 12.46

c) the probability that a household views television more than 5 hours a day is expressed as

P(x > 5) = 1 - P(x ≤ 5)

For x = 5

z = (5 - 8.35)/2.5 = - 1.34

Looking at the normal distribution table, the probability corresponding to the z score is 0.0901

Therefore,

P(x > 5) = 1 - 0.0901 = 0.9099

2x + 3y = 12
Complete the missing value in the solution to the equation.
1,8)

Answers

Answer:

Step-by-step explanation:

[tex]\mathrm{Slope-Intercept\:form\:of}\:2x+3y=12:\quad y=-\frac{2}{3}x+4\\\mathrm{Domain\:of\:}\:-\frac{2}{3}x+4\::\quad \begin{bmatrix}\mathrm{Solution:}\:&\:-\infty \:<x<\infty \\ \:\mathrm{Interval\:Notation:}&\:\left(-\infty \:,\:\infty \:\right)\end{bmatrix}\\\mathrm{Range\:of\:}-\frac{2}{3}x+4:\quad \begin{bmatrix}\mathrm{Solution:}\:&\:-\infty \:<f\left(x\right)<\infty \\ \:\mathrm{Interval\:Notation:}&\:\left(-\infty \:,\:\infty \:\right)\end{bmatrix}[/tex]

[tex]\mathrm{Parity\:of}\:-\frac{2}{3}x+4:\quad \mathrm{Neither\:even\:nor\:odd}\\\mathrm{Axis\:interception\:points\:of}\:-\frac{2}{3}x+4:\quad \mathrm{X\:Intercepts}:\:\left(6,\:0\right),\:\mathrm{Y\:Intercepts}:\:\left(0,\:4\right)\\\mathrm{Inverse\:of}\:-\frac{2}{3}x+4:\quad -\frac{3x-12}{2}\\\mathrm{Slope\:of\:}-\frac{2}{3}x+4:\quad m=-\frac{2}{3}[/tex]

(HALP PLS WILL 100% MARK BRAINLIEST!) Sage has $26 and wants to download some songs from a music
Web site. Each song costs $1.79 to download. By rounding the
cost of the songs up to the nearest whole dollar, estimate the
greatest number of songs she can download.
A) 11
B) 13
C) 15
D) 17

Answers

Answer:

≈ 13

Step-by-step explanation:

First ,we round 1.79 to the nearest whole dollar:

since 7 ≥ 5 then 1.79 rounded to the nearest whole dollar is 2

now ,we can estimate the greatest number she can download:

26/2 = 13

An electronic system has each of two different types of components in joint operation. Let X and Y denote the lengths of life, in hundreds of hours, for components of type I and type II, respectively. The performance of a component is independent for each other. Let E(X) = 4, E(Y) = 2, E(X2) = 24, E(Y2) = 8.
The cost of replacing the two components depends upon their length of life at failure and it is given by C = 50 + 2X + 4Y.
(i) Compute the average cost of replacing the two components. Your final answer must be a number.
(ii) Compute the standard deviation of cost of replacing the two components. Your final answer must be a number.

Answers

Answer:

a

The average cost is [tex]E(C) = 66[/tex]

b

The standard deviation of  cost is  [tex]\sigma = 9.798[/tex]

Step-by-step explanation:

From the question we are told that

          [tex]E(X) = 4[/tex]

          [tex]E(Y) = 2[/tex]

          [tex]E(X^2) = 24[/tex]

           [tex]E(Y^2) = 8[/tex]

The cost of replacing the two component is  C =  50 + 2 X + 4 Y

The variance   of X is mathematically represented as

           V(X) =  [tex]E(X^2) - [E(X)]^2[/tex]

Substituting values  

              [tex]V[X] = 24 - 4^2[/tex]

               [tex]V[X] =8[/tex]

The variance  of Y is mathematically represented as

           V(Y) =  [tex]E(Y^2) - [E(Y)]^2[/tex]

Substituting values  

              [tex]V[Y] = 8 - 2^2[/tex]

               [tex]V[X] =4[/tex]

The average of  replacing the two component is  

        [tex]E(C) = 2 * E(X) + 4* E(Y)[/tex]

substituting value

         [tex]E(C) = 50 + 2 * (4) + 4* (2)[/tex]

         [tex]E(C) = 66[/tex]

The variance  of replacing the two component is  

        [tex]V(C) = V(50 + 2X +4Y)[/tex]      Note: The variance of constant is zero

                                                                  and X and  Y are independent  

=>      [tex]V(C) = 2^2 * V(X) + 4^2 * V(Y)[/tex]

substituting values

=>       [tex]V(C) = 4 * 8 + 16 * 4[/tex]        

=>         [tex]V(C) = 32 + 64[/tex]

=>         [tex]V(C) = 96[/tex]

The standard deviation is

         [tex]\sigma = \sqrt{V(C)}[/tex]  

substituting values

           [tex]\sigma = \sqrt{96}[/tex]

           [tex]\sigma = 9.798[/tex]

Alicia made her aunt a garden. The width is 6 5/7 ft ft and the length is 7 3/5 ft. What is the area of the garden
Please answer!

Answers

Step-by-step explanation:

Area=Length×width

=6 5/7×7 3/5

change the improper fraction to proper

=47/7 ×38/5

=1786/35

=51.03ft²

The solution of the system of equations Ax+y=5 and Ax+By=20 is (2,-3). What are the values of A and B

Answers

Answer:

A = 4B = -4

Step-by-step explanation:

Put the values of x and y in the equations and solve the resulting system.

  A(2) +(-3) = 5

  A(2) +B(-3) = 20

__

The first equation tells the value of A:

  2A = 8 . . . . add 3

  A = 4 . . . . . .divide by 2

This and the second equation tells the value of B:

  (4)(2) -3B = 20

  -3B = 12 . . . . . subtract 8

  B = -4 . . . . . . . divide by -3

The values of A and B are 4 and -4, respectively.

Explain how you will solve the pair of equations by substitution.show all steps and write the solution in (x,y) form.

Answers

Answer:

(-2, -3)

Step-by-step explanation:

The question is incomplete. Here is the complete question.

Consider the following pair of equations:

y = 3x + 3 ... 1

y = x − 1 ... 2

Explain how you will solve the pair of equations by substitution. Show all the steps and write the solution in (x, y) form .

From both equations, we will substitute y = 3x+3 in equation 1 into 2 to have;

3x+3 = x-1

collecting the like terms;

3x-x = -1-3

2x = -4

Dividing both sides by 2 will give;

2x/2 = -4/2

x = -2

Substituting x = -2 into equation 1;

y = 3x+3

y = 3(-2)+3

y = -6+3

y = -3

The solution in (x, y) form is (-2, -3)

A box contains five keys, only one of which will open a lock. Keys are randomly selected and tried, one at a time, until the lock is opened (keys that do not work are discarded before another is tried). Let Y be the number of the trial on which the lock is opened. Find the probability function for Y.

Answers

The "density of probability" is represented by the Y-axis in the normal distribution.It indicates the likelihood of finding values close to comparable places on the X-axis.

Find the probability f ?

Y is the number of trial on which lock is opend so, P(Y=y) be the probability that key opens  the lock on y th trial Lets draw first key

p(1 st key opens the lock) =1-1/5= 4/5

Hence p(1 st key does not open the lock  = 1- 1/5 =4/5

Since  p (Lock is opened on 1 st triual ) = p(1st key opens the lock )

Hence , P(Y =2 ) 1/4 .4/5 = 1/5

p( y = 4) = 1/5

p(y = 5 ) =1/5

To learn more about probability function refer

https://brainly.com/question/28021875

#SPJ2

Answer with Explanation please! Thank you!

Answers

Answer:

D

Step-by-step explanation:

2(x+b) = ax+c

2x+2b = ax+c

c = 2b

PLEASE ANSWER I NEED THIS NOW The blue dot is at what value on the number line?

Answers

Answer:

-10

Step-by-step explanation:

goes up by 4 each value

Answer:

-10

Step-by-step explanation:

in that chain,

it is 10,6,2,-2,-6,-10

Help, pls. it's due tomorrow night.
An airplane flies due north at 200 km/h, and the wind blows in a direction N30°E
at 40 km/h. Find the coordinates for the vector representing the resultant for
the airplane with the wind factored in, and the resultant airspeed. Report any
approximations to two decimal places accuracy and include a diagram.​

Answers

Answer:

please please please please show please

Suppose P(x) represents the profit on the sale of x Blu-ray discs. If P(1,000) = 5,000 and P'(1,000) = −3, what do these values tell you about the profit? P(1,000) represents the profit on the sale of Blu-ray discs. P(1,000) = 5,000, so the profit on the sale of Blu-ray discs is $ . P'(x) represents the as a function of x. P'(1,000) = −3, so the profit is decreasing at the rate of $ per additional Blu-ray disc sold.

Answers

Answer:

Step-by-step explanation:

We are told that P(x)  is the profit of saling x blu ray discs. P(1000) is our profit for selling 1000 blu ray discs. So, our profit is 5000. Recall that the derivative P'(x) represents the rate at which the function P(x) is increasing/decreasing (increasing if P'(x) is positive, or decreasing otherwise) by increasing the values of x. In this case P'(1000)=-3, so the profit will decrease -3 if we increase x in one unit.

The radius of a circle is 6.5cm What is the diameter​

Answers

The diameter of a circle is twice the radius.

Diameter = radius x 2

                = 6.5 x 2

                = 13 cm

Answer:

13

Step-by-step explanation:

diameter is two times the radius

6.5 x 2 = 13

Anja stands by the side of the road counting the wheels on the vehicles that go past her if she counts 250 wheels tie many cars and bikes has she seen?

Answers

Answer:

50 cars and 25 bikes

Step-by-step explanation:

Given

Total wheels = 250

Required:

Number of cars and bikes

Let C represent Cars and B represents Bike

A car has 4 wheels and a bike has 2 wheels;

So,

[tex]4C + 2B = 250[/tex]

Divide through by 2

[tex]\frac{4C + 2B}{2} = \frac{250}{2}[/tex]

[tex]2C + B = 125[/tex] ---- Equation 1

The ratio of wheels of cars to wheels of bike is 1:2

Meaning that

1C = 2B

So, C = 2B

Substitute 2B for C in equation 1

[tex]2(2B) + B = 125[/tex]

[tex]4B + B = 125[/tex]

[tex]5B = 125[/tex]

Divide both sides by 5

[tex]\frac{5B}{5} = \frac{125}{5}[/tex]

[tex]B = \frac{125}{5}[/tex]

[tex]B = 25[/tex]

Recall that C = 2B

So,

[tex]C = 2 * 25[/tex]

[tex]C = 50[/tex]

Hence, Anna has seen 50 cars and 25 bikes

Bartholomew went hiking over the weekend. He hiked all 4 trails in 3 hours. Which is the best estimate for the number of miles he hiked per hour

Answers

Answer:

a?

Step-by-step explanation:

Last week, the price of cherries at the corner deli was $4.99 per pound. This week, cherries at the same deli are on sale at a 15% discount. What is the total price of 3.75 pounds of cherries this week at the deli?

Answers

Answer:

The price for 3.75 pounds of cherries is $17.775.

Step-by-step explanation:

The price las week was $ 4.99 and it got a discount of 15% for this week, this means that the price for this week is 100% of the one prior minus 15%, therefore it is 85% of the last price. We need to calculate the price per pound with this discount as shown below:

[tex]\text{this week} = \text{last week}*\frac{95}{100}\\\\\text{this week} = 4.99*0.95 = 4.74 \text{ per pound}\\[/tex]

Since the price is $4.74 per pound and we want to buy 3.75 pounds, then the total amount we need to pay is:

[tex]\text{total amount} = 3.75*4.74 = 17.775[/tex]

The price for 3.75 pounds of cherries is $17.775.

Cuanta cartulina se necesita para construir un prisma rectangular para cuya base mide 10cm y tiene 6cm de altura

Answers

Answer:

i dont understand the language

Step-by-step explanation:

which side lengths form a right triangle

Answers

Answer:

A and C

Step-by-step explanation:

A right triangle consists of two legs and a hypotenuse.To find hypotenuse, we use The Pythagorean TheoremThe Pythagorean Theorem = [tex] {a}^{2} + {b}^{2} = {c}^{2} [/tex]two legs meet at a 90° angle and the hypotenuse is the longest side

3 - 4 - 5

6 - 8 - 10

7 - 24 - 25

5 - 12 - 13

are some basic right triangle

(about more, look picture)

Just use hypotenuse formula :

3 x 3 = 9

4 x 3 = 12

5 x 3 = 15

Option C is correct

[tex] {5}^{2} + { \sqrt{6} }^{2} = { \sqrt{31} }^{2} [/tex]

Option A is correct

[tex] {5}^{2} + {5}^{2} = { \sqrt{50} }^{2} [/tex]

But B isn't correct

Hope this helps ^-^

A 6) Set both given equations equal to zero, then combine them into one standard form
equation. Simplify if possible.
7x + 3 = 5 and y-1 = 6

Answers

The equation in standard form is given by:

[tex]7x + y - 9 = 0[/tex]

------------------

The general equation in standard form is:

[tex]Ax + By + C = 0[/tex]

------------------

The first equation is:

[tex]7x + 3 = 5[/tex]

Setting equal to zero:

[tex]7x + 3 - 5 = 0[/tex]

[tex]7x - 2 = 0[/tex]

------------------

The second equation is:

[tex]y - 1 = 6[/tex]

[tex]y - 1 - 6 = 0[/tex]

[tex]y - 7 = 0[/tex]

------------------

Combining them, and placing in standard form:

[tex]7x - 2 + y - 7 = 0 + 0[/tex]

[tex]7x + y - 9 = 0[/tex]

A similar problem is given at https://brainly.com/question/14664782

What is the greatest common factor of 60, 45 and 82? HALPPPPPPPPPPPPPPPPPPp

Answers

Answer:

GCF: 1

Step-by-step explanation:

Factors are numbers that can be evenly divided into another number. The greatest common factor of a set of numbers is the biggest number than can evenly divide into all the the numbers.

Let's list the factors of 60, 45 and 82

Factors of 60:

1, 2, 3, 4, 5, 6, 10, 12, 15, 20, 30, 60

Factors of 45:

1, 3, 5, 9, 15, 45

Factors of 82:

1, 2, 41, 82

The greatest factor that all 3 numbers share is 1.

The right answer is 1.

Please see the attached picture for full solution

Hope it helps

Good luck on your assignment..

Is the integral ſ A. dr path independent if A = (2x - y)i + (x + y)j?

Evaluate A dr around a unit circle centered at the origin. What is t

value of the integral if we take a clockwise direction?

Answers

The integral is path-independent if we can find a scalar function f such that grad(f ) = A. This requires

[tex]\dfrac{\partial f}{\partial x}=2x-y[/tex]

[tex]\dfrac{\partial f}{\partial y}=x+y[/tex]

Take the first PDE and integrate both sides with respect to x to get

[tex]f(x,y)=x^2-xy+g(y)[/tex]

where g is assumed to be a function of y alone. Differentiating this with respect to x gives

[tex]\dfrac{\partial f}{\partial y}=-x+\dfrac{\mathrm dg}{\mathrm dy}=x+y\implies\dfrac{\mathrm dg}{\mathrm dy}=2x+y[/tex]

which would mean g is *not* a function of only y, but also x, contradicting our assumption. So the integral is path-dependent.

Parameterize the unit circle (call it C) by the vector function,

[tex]\mathbf r(t)=\cos t\,\mathbf i+\sin t\,\mathbf j[/tex]

with t between 0 and 2π.

Note that this parameterization takes C to have counter-clockwise orientation; if we compute the line integral of A over C, we can multiply the result by -1 to get the value of the integral in the opposite, clockwise direction.

Then

[tex]\mathrm d\mathbf r=-\sin t\,\mathbf i+\cos t\,\mathbf j[/tex]

and the (counter-clockwise) integral over C is

[tex]\displaystyle\int_C\mathbf A\cdot\mathrm d\mathbf r[/tex]

[tex]\displaystyle=\int_0^{2\pi}((2\cos t-\sin t)\,\mathbf i+(\cos t+\sin t)\,\mathbf j)\cdot(-\sin t\,\mathbf i+\cos t\,\mathbf j)\,\mathrm dt[/tex]

[tex]\displaystyle=\int_0^{2\pi}1-\sin t\cos t\,\mathrm dt=2\pi[/tex]

and so the integral in the direction we want is -2π.

By the way, that the integral doesn't have a value of 0 is more evidence of the fact that the integral is path-dependent.

Other Questions
PLEASE HELP!!!!!!!!!!!!!!!!!!!!!!!!Hola, amigos. Mi nombre es Bianca. El ao pasado visit La Mancha, en Espaa, y alquil una cabaa con internet, cocina, jardn y reas de camping y pcnic. La cabaa tena agua caliente cada da. Caminbamos 10 minutos al centro turstico para comprar artculos de aseo personal. Este ao, mi familia y yo pasamos una semana en una cabaa en Carolina del Norte (North Carolina). La cabaa tena artculos de aseo personal, pero solo tena internet de las 6 de la maana a las 11 de la noche. Tenamos un rea de pcnic donde cocinbamos y jugbamos cada da. Based on the text, what is similar about these two cabins? Access to hot water Access to picnic areas Unlimited access to camping areas Unlimited access to internet evaluate -5+(-5r)+10 150 miles converted to kilometres? Solving for Unknown AnglesLines x and y are parallel.Parallel lines x and y are intersected by lines s and t. At the intersection of lines x, t, and s, clockwise from top left, the angles are blank, blank, (6 x + 8) degrees, blank, (7x minus 2) degrees, blank. At the intersection of lines x and y, the angles are 106 degrees, 1, blank, blank. At the intersection of lines y and t, the angles are 2, blank, blank, blank.Use the diagram to determine the measure of the unknown angle.m1 = (6x + 8)m1 = 74What is the value of x?What is the measure of 2?(7x - 2) + m2 = 106m2 = Buckeye Incorporated had the following trial balance at the beginning of November.BUCKEYE INCORPORATEDTrail BalanceAccounts Debits CreditsCash $ 2,800Accounts Receivable 800Supplies 1,200Equipment 9,000Accounts Payable $ 1,500Notes Payable 2,100Common Stock 5,100Retained Earnings 5,100--------------------------------------------------------------------------------The following transactions occur in November:November 1 Issue common stock in exchange for $13,000 cash.November 2 Purchase equipment with a long-term note for $4,300 from Spartan Corporation.November 4 Purchase supplies for $2,000 on account.November 10 Provide services to customers on account for $10,000.November 15 Pay creditors on account, $2,000.November 20 Pay employees $3,500 for the first half of the month.November 22 Provide services to customers for $12,100 cash.November 24 Pay $3,000 on the note from Spartan Corporation.November 26 Collect $5,000 on account from customers.November 28 Pay $1,900 to the local utility company for November gas and electricity.November 30 Pay $4,600 rent for November.Record each transaction in a general journal All parts of the heart Describe the plot of Charlie and the chocolate factory novel? Laura received a $90 gift card for a coffee store. She used it in buying some coffee that cost $8.63 per pound. After buying the coffee, she had $55.48 left onher card. How many pounds of coffee did she buy? The cell membrane serves many functions. One of the cell membrane's functions is to help the cell maintain homeostasis. Which of the following statements best supports this claim?A. The cell membrane contains phospholipids.B. The cell membrane contains a polar region and a nonpolar region.C. The cell membrane regulates what goes in and out of the cell.D. The cell membrane contains proteins. pls help!! A 24.0 mL sample of HNO3 is neutralized by 32.1 mL of 0.150 M KOH. What is the molarity of HNO3? Report your answer with 3 significant figures. Which of the following wouldmake a good thesis statement onthe topic of helping thehomeless?A. Much of the time and money we spend onsaving whales should be spent on saving people.B. What can we possibly do to better help themany homeless people in our cities?C. It would be nice if we could figure out someway to take care of homeless people in our cities.D. There have been numerous studies on thecause of homelessness in America and where itoccurs. OMG THIS QUESTION IS SO HARD WILL RATE IF U GET IT Which excerpt from the song, Brother, Can You Spare a Dime? best emphasizes the idea that average Americans felt abandoned by the wealthy elite and the government? They used to tell me I was building a dream, And so I followed the mob When there was earth to plow, or guns to bear I was always thereright on the job. Once I built a railroad, made it run, Made it race against time. Once I built a railroad, Now it's done Say, don't you remember, they called me Al It was Al all the time. Why don't you remember, I'm your pal Buddy, can you spare a dime? Once I built a tower, up to the sun, Brick, and rivet, and lime, Once I built a tower, Now it's done Mark this and return what is the specific location of cyprus What is the chemical formula for the compound formed between sodium and sulfur? Please answer this correctly A=8x10 B=3x+90 PLEASE PLEASE HELP OR I WON'T PASS MY MATH CLASS AND I WILL HAVE TO TAKE ANOTHER YEAR OF MATH Nelson Industries warrants its products for one year. The estimated product warranty is 4.3% of sales. Sales were $475,000 for September. In October, a customer received warranty repairs requiring $215 of parts and $65 of labor. Required: 1. Journalize the adjusting entry required at September 30, the end of the first month of the current year, to record the estimated product warranty expense. 2. Journalize the entry to record the warranty work provided in October. If an amount box does not require an entry, leave it blank. A loop of wire is placed into a magnetic field between two magnets. Since this is a three dimensional, a dashed line is drawn through the loop to show the imaginary slice we take that allows us to picture the electron current flow from different perspectives. How do you think the loop will move? Describe this from the front view perspective a) clockwiseb) counter-clockwisec) upd) down what is the tangent ratio of E L?A. 3/4B. 5/3 C. 5/4D. 4/3